Which number produces a rational number when added to 0.5?
A. –1.73205081…
B. 1/4
C. 0.54732871…
D. √3

Answers

Answer 1

Answer:

Option B is correct 1/4.

Step-by-step explanation:

To find : Which number produces a rational number when added to 0.5?

Solution :

A rational number is defined as number written in simple fraction or in p/q form.

An irrational number is defined as number which cannot be written in p/q form.

We know that,

A rational number adding a rational number gives you a rational number.

A rational number adding an irrational number gives you a irrational number.

Now we take each option and check.

Option A : -1.73205081…

Number has no end and have infinite number of digits so it is irrational number.

No, this number not produce a rational number.

Option B : 1/4=0.25

Number is in form p/q so it is rational number.

Yes, this number produce a rational number.

0.25+0.5=0.75 a rational number.

Option C : 0.54732871…

Number has no end and have infinite number of digits so it is irrational number.

No, this number not produce a rational number.

Option D : \sqrt{3}

\sqrt{3} cannot be written in ratio so it is irrational number.

No, this number not produce a rational number.


Related Questions

If a car has already traveled 10 miles and then continues for another 6 hours at a steady rate of 30 miles per hour . how many miles will it travel? ANSWER PLZ

Answers

Answer:

190 miles

Step-by-step explanation:

Using distributive property 49×17+49×3=​

Answers

Answer:

980

Step-by-step explanation:

➡ 49 × 17 + 49 × 3

➡ 49 ( 17+3 )

➡ 49 × 20

➡ 980

Answer : 980

Which graph represents a function with a rate of change of 0.5?
5
1
-5 -4 -3 -2
4
5
Ty
5
4
3
2
1

Answers

It’s 5 because the other ones not important

help me please!!!!!!!!!!!!​

Answers

Answer:

?

Step-by-step explanation:

Please help this fairly quick

Answers

Answer:

A) true

B) false

c) true

d) true

Step-by-step explanation:

A) angles 7 and 6 are vertical angles, which means they're equal.

so, A is True.

B) angles 7 and 5 are supplementary angles, which means they add up to 180. 135 + 55 is 190. so B is False

C) angle 7 is equal to angle 3, which is supplementary to angle 1.

so, 55 + angle 1 = 180

55 + angle1 = 180

subtract 55 from both sides of the equation

angle1 = 180 - 55

angle 1 = 125.

angles a and 4 are vertical angles, so they are equal. if you add them together that would be 125 + 125

125 + 125 = 250

so, C is True

D) angles 1 and 6 are supplementary, and angles 2 and 7. since they both add together to make 180, D is True

Pleaseeeee explain why these faces are or are not congruent in two sentences, with two pieces of evidence.

Answers

The answer would be: These two faces are not congruent. The right face’s eyes are father apart and the mouth is lower than the left face.

hat is the speed of a wave if the wavelength is 18 m and the frequency is 35 Hz? (Quick Hurry, need answer)

Answers

The speed is 630mp/s I think. Because you have to multiply Wavelength by Frequency to get true speed

which of the following expressions are equivalent to (6.2+6.2)+9.4? Choose two answers. plz help​

Answers

Answer:

Step-by-step explanation:

In my opinion the answer should be A B and E

-35+7k = -7+7(k-4)
is it IMS (Infinite solutions)
one solution
no solution?​

Answers

Answer:

-35+7k=-7+7 (k-4)-42=-14 (k-4).This the simplest way to simply the work.

How would you explain the following problem to a friend who didn't understand today's lesson?
Three angles of a triangle measure 10x+15, 8x+3 ,and 6x–6. Find the measure of the smallest angle.

Answers

Answer:

The smallest angle is 6x-6 which is 36°

Please helpppppppppppppppppppppppppppppppppppppppppppppppppppppppp

Answers

Answer:

Here you go :)

Hey, don't sweat it, this is relatively simple.

0 = $0

1 = $400

2 = $800

3 = $1400

________

Answered by James

Step-by-step explanation:

Just keep adding my friend

Which choice best represents the height an airplane flies?
A)10 cm
B)10m
C)10mm
D)10km

Answers

Answer:

I think 10 km.

Please mark as brainliest.

Answer: 10 kilometers

10 kilometers is about 38,500 feet. This is the average altitude an average commercial plane should fly. The answer to this question is D, 10 kilometers.

Hope this helps!

Lance is solving the equation 2(2x + 1) - (3x - 5) = 17. His steps are shown below: Step 1: 4x + 2 - (3x - 5) = 17 Step 2: 4x + 2 - 3x - 5 = 17 Step 3: x + 2 - 5 = 17 Step 4: x - 3 = 17 Did Lance make a mistake? If so, in which step did he make the mistake and what was his mistake?

Answers

Answer:

Lance made a mistake

His mistake was in step 2

Step-by-step explanation:

Yes, Lance made a mistake.

He made a mistake in step 2 of his solution.

The mistake he made was that he was supposed to open the bracket of -(3x-5) to give -3x + 5

The 5 is supposed to have a positive value and not a negative

y = 3x - 2
x - y = 4

Answers

Answer:

x=-1, y=-5

Step-by-step explanation:

y=3x-2 (1)

x-y=4 (2)

sub (1) into (2)

x-(3x-2)=4

x-3x+2=4

-2x=2

x=-1

sub x=-1 into (1)

y=3(-1)-2

 =-3-2

y = -5

Therefore: x=-1, y=-5 :))

Answer:

x = -1, y = -5

Step-by-step explanation:

y = 3x - 2

x - y = 4

x - y = 4

x - (3x - 2) = 4

x - 3x + 2 = 4

-2x + 2 = 4

-2x = 2

x = -1

x - y = 4

-1 - y = 4

-y = 5

y = -5

Check:

x - y = 4

-1 - (-5) = 4

-1 + 5 = 4

4 = 4

1) A recipe for cookies calls for 4 cups of flour and 2 cups of sugar. What is
the amount of flour and sugar needed for 1,2,3,4 and 5 batches of cookies

Answers

Answer:

4:2 8:4 12:6 16:8 20:10

Step-by-step explanation:

4:2 is the amount needed for each batch so you can multiply that amount by the number of batches.

i want to get at least a 90 on my next math test

Answers

Answer:

then study well

Step-by-step explanation:

that might help, but I could help with any study questions, but I cant do test questions from the test

Make sure to study any books that they give you trust me it will help

Can you help me solve for a?

Answers

Answer:

10

Step-by-step explanation:

We can look at the denominators(bottom numbers in a fraction), and find a relationship. 15 is 5 times 3, so let's multiply 2 by 5 to get z!

2(5)=10

(? x 3) - 3 = 15

help pls and i need the steps :/​

Answers

Answer:

4

Step-by-step explanation:

Lets make [tex]x[/tex] be the ? in the equation.

That becomes ([tex]x[/tex] x 3) - 3 = 15.

First you have to add 3 on both sides. After doing that it gets to ([tex]x[/tex] x 3) = 15 - 3.

Then you simplify it to 3[tex]x[/tex] = 12.

Now you divide 3 on both sides. That gets to [tex]x[/tex] = 4 after doing it.

Hope this helped! If not, please let me know! <3

Jack plans to buy 2.25 lb of coleslaw at $2.80 per pound. He also wants 2lb of macaroni salad that sells for $3.15 per pound. Jack has a $10 bill in his wallet. Is this enough for the coleslaw and the macaroni? Explain.

Answers

No

2.25 times 2.80 gives the cost for the total coleslaw, which is $6.30
2 times 3.15 gives the cost for the total macaroni salad, which is also $6.30
Add those together and you get $12.60, which is larger than $10, so it is not enough
First, multiply 2.25 by 2.80 to get the cost of the coleslaw, which happens to be $6.30. Then, multiply 3.15 times 2, which would also get $6.30. Last, add $6.30 + $6.30, to get $12.60, which is more than $10, so he doesn’t have enough.

PLS HELP FAST if all three angles of a triangle are 60 degrees, what are the length of the side(s)?

Answers

Answer:

180

Step-by-step explanation:

the lengths are all 180 because 60 degrees times three sides is 180 so the length of one side is 180.

What is the.least common multiple of 2 and 6

Answers

Answer:

2

Step-by-step explanation:

Answer:

The answer to this is: The Least Common Multiple of 2 and 6 is 6.

Step-by-step explanation:

Hope this helps :)

(4, 10) and (-2,-5) Slope​

Answers

Answer:

The slope is 5/2

for each ordered pair, determine whether it is a solution to 7x+3y=2

the ordered pairs:
0, -8
-1, 3
2, -4
5, 2

Answers

Answer:

Step-by-step explanation: i thimk 5,2

John is a quarterback. This year, he completed 350 passes, which is 70% of all the passes he's attempted this year. How many passes has John attempted this year? passes​

Answers

Answer:

500 passes

Step-by-step explanation:

multiply 350 by 100 and divide by 70

350x100=35,000

35,000/70

=500

and this is funny cuz my brother plays football and a QB and his name is john. creepy times

Over a period of 12 hours the temperature dropped 36oF. What was the average change in temperature in one hour?

Answers

Answer:

The average change of temperature in one hour is -3

Step-by-step explanation:

You had to divide 36 by 12

Use an area model to solve 2 2/3 x 4 35.

Answers

Answer:

I can give you the answer right of the top of my head here it is 1 and 3/4 x 2 and 1/2 is 4 and 3/8

Step-by-step explanation:

Make me brainlist plsssss

solve for x if z = m - x

Answers

Answer:

x=m-z

Step-by-step explanation:

The first step is to isolate x and to do this we got to subtract m from both sides which would give us -m+z=-x, the next step is to get x to be positive so we have to divide both sides by negative 1 which is because there is a 1 beside all of the values, and when we do divide both sides by negative 1 we get x=m-z

Use the diagram and the given angle measure to find the other three measures.

m∠1=143∘

m∠2=

m∠3=

m∠4=

Answers

Answer:

angle 2 = 37⁰

angle 3 = 153⁰

angle 4 = 37⁰

Step-by-step explanation:

I guess this refers to two intersecting lines which form 4 angles.

Angles 1 and 2 are supplementary angles whose sum is 180⁰.

Angle 1 + angle 2 = 180⁰

143⁰ + angle 2 = 180⁰

angle 2 = 180⁰ - 143⁰

angle 2 = 37⁰

Angles 1 and 3 are vertical angles so do with angles 2 and 4. Therefore it is easy to find their measures because vertical angles are congruent in measures.

Hope this helps...

Answer:

m∠1 = 143°

m∠2 = 37°

m∠3 = 143°

m∠4 = 37°

SOMEONE PLEASE HELP I NEED THE SOLUTION TO ALL OF THESEE PLEASE HELP!!!!

Answers

Answer:

The answer is below

Step-by-step explanation:

Let's work it out slowly

A)

$18.79 + $2.11 - $1.92 + $17.28

- 18.79 + 2.11 + 17.28 = 38.18

- 38.18 - 1.92 = 36.26

Ans = $36.26

B)

$7.45 + $24.45 + $74.17 + $76.52

- 7.45 + 24.45 = 31.9

- 74.17 + 76.52 = 150.69

- 150.69 + 31.9 = 182.59

Ans = $182.59

C) $98.45 - $10.63 + $2.82 - $20.26

- 98.45 - 10.63 - 20.26 = 67.56

- 67.56 + 2.82 = 70.38

Ans = $70.38

Hope you understand

:-)

The answer to A is $40.1
The answer to B is $-19.35
The answer to C is $70.38
Hope this helps.

PLEASE HELP ME ASAP!!! I will name you Brainliest!

Answers

Answer:

I think its (x+1)+(x+3)+(x+5)=63

Step-by-step explanation:

you add all the ages together not multiply so that gets rid of D and it says odd numbers which will get rid of a and c

Other Questions
Settlers in the backcountry created new distinctive cultures, fueling social and ethnic tensions and often resulting in violent protest to express grievances."For TWO of the following, analyze the causes and effects of the revolt: *Bacon's Rebellion *Shay's Rebellion *Whiskey Rebellion Explain which ONE event best reflects the statement above. Explain why your choice is better than ONE of the other choices listed. PLEASE HELP!!! Georgia held eight land lotteries between 1805 and 1833, showing that Sharia law is based on the Islamic holy book, called the __________.A.KnessetB.MajlisC.QuranD.SultanPlease select the best answer from the choices providedABCD According to The Declaration of Independence, what is the main purpose of government?(1 Punto)To provide for common defense and promote the general welfareTo create a strong national governmentTo protect the interests of the statesTo protect citizens rights to life, liberty, and the pursuit of happinessquotation to ansamong Ment What's the most likely way that water might have been lifted to the terracesof the Hanging Gardens? Please help me with this band assignment the product of 120 and b is equal to 312Write the sentence as an equation. a _________ reaction occurs when you mix an acid with a base liniar systems3x-2y=-72x+5y+=8 From the list below, choose five activities you do a lot.Write your activities in the oval labeled Yo. Be sure toconjugate the verb in the yo form.trabajarcantorbailarnadar estudia James and Tyree have new jobs at YumYums Ice Cream Parlor. James is Tyrees manager. In their first year, James will be paid $14 per hour and Tyree will be paid $7 per hour. They have been told that after every year with the company, they will each be given a raise of $2 per hour. Is this realationship porprotional? What does Paul list as the core components of a church service? Why are these important and central? What is a difference between systemic and pulmonary circulation? Help !!!!! Thank youuu Using the table on the right, perform the following metric unit conversions. Type your answer in the space provided.0.001087 kg = g what statement is best supported by the data shown how do i solve this? Who was the most famous ruler of Mali what what effect did he have on the North African economy What element of tone is unique to a stage production?O A. PlotO B. Word choiceO C. ConflictO D. Sound HELP PLEASE!!!!!!!!Consult the poem to help you answer the question."The Tyger" by William Blake Tyger Tyger, burning bright, In the forests of the night; What immortal hand or eye, Could frame thy fearful symmetry? In what distant deeps or skies Burnt the fire of thine eyes! On what wings dare he aspire? What the hand, dare seize the fire? And what shoulder, & what art, Could twist the sinews of thy heart? And when thy heart began to beat, What dread hand? & what dread feet? What the hammer? what the chain, In what furnace was thy brain? What the anvil? what dread grasp, Dare its deadly terrors clasp? When the stars threw down their spears And waterd heaven with their tears: Did he smile his work to see? Did he who made the Lamb make thee? Tyger, Tyger burning bright, In the forests of the night: What immortal hand or eye, Dare frame thy fearful symmetry?In "The Tyger," what conclusion can be drawn about the tigers creator?